You are on page 1of 70

LESSON 1- REFRACTION OF

LIGHT
1.1 Introduction 1.3 Refractive index,n 

Refraction of light is a phenomenon where the The refractive index is a determinant of the
direction and speed of light  are changed ( change sterength of the bending of light when light rays
in velocity) when it crosses the boundry between travel from vacuum to a medium.
two materials of different optical densities.
                           1.3.1. The definition of the refractive index
1.2 Three ways in which a ray can travel          
through two  mediums The refractive index is defined as:

1 When a ray of light travels from optically less


dense medium to optically denser medium , the speed of light in vacuum
ray is bent towards  the normal. Refractive index 
speed of light in medium
After entering the  denser medium the speed of
light decreases and angle of incidence > angle c
n
of refraction. v
c = 3 x 108 m s-1

1.3.2. Relationship between refractive index


and optical density of medium

The optical density of a medium is not the same


2 When a ray of light travels from optically as its physical density
denser medium to optically less dense medium , Optical density determines how much a light
the ray is bent  away from the normal. wave is slowed down as it passes through a
After entering the less dense medium the speed medium. The more optically dense a material is,
of light increases and angle of incidence < angle the slower that a wave will move through the
of refraction. material. The refractive index is a measurement
      of optical density.
The following table shows the refractive index
for some media.
Medium Refractive index, n
Vacuum and air 1.00
Ice 1.31
Water 1.33
Alcohol 1.38
Olive oil 1.46
3 When a ray of light crosses the boundry Parafin 1.48
between two different mediums at a right angle Perspex 1.50
or the incident ray parallel to normal, the ray is Glass (crown) 1.52
not bent but the  speed of light is different. Glass (flint) 1.66
The speed of light decrease when ray travel Diamond 2.42
from less dense medium to denser medium and Refractive index ≥ 1 and has no unit.
the speed of light increase when the ray of light From the table above , we can make a
travel from denser medium to less dense conclusion that the higher the optical density a
medium. medium , the higher the refractive index and the
The angles of incidence and refraction are zero. smaller the bending.

Optical density : ρ1 < ρ2 < ρ3


Refractice index : n1 < n2 < n3
Angle of refraction : θ1 > θ2 > θ3

1
Speed of light : v1 > v2 > v3 sin i
1.4 Law of refraction n
sin r
(2) When ray of light travel from medium 1
to medium 2 or vice-versa :

1.4.1 First law :


Use the formula :
The incident ray , normal and the n1 sin θ1 = n2 sin θ2
refracted ray all lie in the same plane

1.4.2 Second law ( Snell’s law): 1.4.3 Experiment to determine the


refractive index of a glass block.
Snell’s law states “the ratio of the sines of the
angles of incidence and sines the angles of Aim of the experiment : 
refraction are constant” when the passes
between two given media. To determine the refractive index of glass
sinus i block.
 constant  n
sinus r
List of apparatus and materials:

Glass block, ray box, white paper 


protractor,power supply 

Arrangement of the apparatus:

Based on the Snell’s law

sin i sin i
 n1 and  n2
sin 1 sin  2

Hence n1 sin θ1 = n2 sin θ2

As the conclusion :

(1) When ray of light travel from air to a


medium or vice-versa :

The procedure of the experiment

The glass block is placed on a white paper.


The outline of the sides of the glass block
are traced  on the white paper and labelled as
ABCD.
The glass block is removed. 
The normal  ON is drawn.
By using a protractor , the angle of incidence
Use the formula : is measured , i = 30o
The glass block is replaced again on its
outline on the paper.

2
A ray of light  from the ray box is directed
along incidence line.
The ray emerging from the side CD is drawn
as line PQ. Arrangement of the apparatus:
The glass block is removed again.
The point  O and P is joined and is drawn as
line OP.
The experiment is repeated 4 times for the
other angles of incidence i = 40o, 50o, 60o
and 70o

Data:

i (o) 30 40 50 60 70
r (o)
sin i
sin r

Calculation:
The procedure of the experiment which
  Plot the graph sin i against sin r
include the method of controlling the
manipulated variable and the method of
measuring the responding variable.

The glass block is placed on a white paper.


The outline of the sides of the glass block are
traced  on the white paper and labelled as
ABCD.
The glass block is removed. 
The normal  ON is drawn.
Determine the gradient of the graph = m By using a protractor , the angle of incidence is
Refractive index , n = m measured , i = 30o
The glass block is replaced again on its outline
1.4.4 Experiment to investigate the on the paper.
relationship between the angle of incidence A ray of light from the ray box is directed along
and the angle of refraction.  incidence line.
The ray emerging from the side CD is drawn as
Hypothesis:  line PQ.
The glass block is removed again.
The angle of refraction increases as the angle of The point  O and P is joined and is drawn as line
incidence increases. OP.
The experiment is repeated 4 times for the other
Aim of the experiment :  angles of incidence i = 40o, 50o, 60o and 70o
To investigate the relationship between the Tabulate the data:
angle of incidence and the angle of refraction.
i (o) 30 40 50 60 70
Variables in the experiment:
r (o)
Manipulated variable: Angle of incidence sin i
Responding variable: Angle of refraction sin r
Constant variable: Refractive index
Analysis the data:
List of apparatus and materials:
 Plot the graph sin r against sin i
Glass block, ray box, white paper 
protractor,power supply 

3
Pin,ruler,water,retort stand ,tall beaker
and metre ruler.

1.6 Real depth (H) and apparent depth (h) Arrangement of the apparatus:

1.6.1 : Meaning real depth and apparent


depth

Real depth, H : The depth of an object


from the surface of
medium
Apparent depth , h : The depth of the image
from the surface of
medium

The procedure of the experiment.

Pin 1 is placed at the base of the beaker as


object O.
Pin 2 is clamped  horizontally onto the retort
stand as image position indicator, I
The beaker is filled with water.
By using a ruler ,the real depth of the pin is
measured ,H = 30 cm
The pin 1is seen vertically above the surface of
the water.
The position of pin 2 is adjusted until the pin
1and the pin 2 seen colinear.
By using the ruler again ,the position of pin 2 is
measured as the apparent depth = h
The experiment is repeated 4 times for the other
value of H= 40 cm , 50 cm , 60 cm and 70 cm

Data of the experiment

H (cm) 30 40 50 60 70
h (cm)
1.6.2 Relationship between n , H and h
Calculation
real depth
Refractive index  Plot the graph H against h
apparent depth
H
n
h
1.6.3 Experiment to determine the refractive
index of water.

. Aim of the experiment : 

To determine the refractive index of water.

List of apparatus and materials:


Determine the gradient of the graph = m

4
Refractive index , n = m The position of pin 2 is adjusted until parallax
error between the pin 1and the pin 2 is non-
existent.
By using the ruler again ,the position of pin 2 is
measured as the apparent depth = h
The experiment is repeated 4 times for the other
value of H= 40 cm , 50 cm , 60 cm and 70 cm
1.6.4 Experiment to investigate the
relationship between real depth and
apparent depth. Tabulate the data:
H (cm) 30 40 50 60 70
Hypothesis: 
h (cm)
The apparent depth  increases as the real depth
Analyse the data:
increases.
Plot the graph h against H
Aim of the experiment : 

To investigate the relationship between apparent


depth  and the real depth

Variables in the experiment:

Manipulated variable: real depth


Responding variable: apparent depth
Constant variable: Refractive index
1.6.5 To complete diagram of refraction of
light
List of apparatus and materials:

Pin,ruler,water,retort stand ,tall beaker 1- Draw image of the object


2- Draw straight line from image to the observer’s eye
Arrangement of the apparatus: 3- Draw normal
4- Draw straight line from normal to objcet
5- Draw arrow from object to observer’s eye

Example 1

 Complete the following ray diagrams:

(a) 

The procedure of the experiment which


include the method of controlling the
manipulated variable and the method of
measuring the responding variable.

Pin 1 is placed at the base of the beaker as


object O.
Pin 2 is clamped  horizontally onto the retort
stand as image position indicator, I
The beaker is filled with water.
By using a ruler ,the real depth of the pin is
measured ,H = 30 cm (b)
The pin 1is seen vertically above the surface of
the water.

5
(c)

(c) 1. 7 To solve some problem involving the


refraction of light

Example 2

Diagram shows a ray of light passing from air to the


block  X.
[ Speed of light in air = 3 x 108 m s-1 ]

Solution:

(a) 

Calculate

(a) The refractive index of the block X.


(b) The speed of light in the block X

Solution

(b)

Example 3

Diagram shows a ray of light passing from a glass


block  to air.
[ Refractive index of glass = 1.52 ]

6
Determine the value of  the angle θ .

Solution (b)

Example 4 Example 6

Diagram shows a ray of light passing from medium Diagram  shows an observer looking at a fish
M to medium N with refractive index s 1.49 and swimming in a pond.
1.36 respectively.

.
Calculate the angle of k. The refractive index of the water is 1.33.
Calculate
Solution (a) the depth of the image of the fish?
(b) the distance between the observer and the image
of the fish.

Solution

Example 5

Diagram shows a light ray NO entering in a glass Example 7


prism.
[ Refractive index of glass = 1.51 ] Diagram shows a man observing the base of a
swimming pool . The base of the pool appear to be
shallower.
The real depth of the swimming pool is 2.0 m.
[ Refractive index of water = 1.33 ]

                              
(a) Why the ray NO does not bent when
entering the glass prism.
(b) Draw the ray light after it passes through a
point P and determine the angle of refraction
when the ray light entering the air again.  (a) What is the apperent depth of the swimming
pool?
Solution (b) In diagram above ,draw  a ray diagram
from point Z to show how the point Z appear
(a) shallower.

7
Solution
(a)

(b)

Which row describes how the speed and the


direction of the ray of light change when it
enters the air?
speed in air direction in air
A decreases towards to the normal
TUTORIAL 1 B decreases away from the normal
C increases towards to the normal
1 When light travels from one medium to another, D increases away from the normal
refraction take place. Refraction is caused by 4 Diagram shows a light ray propagating from air
the change in the  into block M.

A amplitude  of light rays


B intensity of light rays
C strength of light rays 
D velocity of light rays

2 Diagram shows a light ray propagating from air


into water. Wwhich of the physical quantity is zero?

A Speed of light
B Angle of incidence
C Optical density of block M
D Refractive index of block M

5 Diagram  shows a light ray ON propagates from


medium 1 into medium 2. The optical density of
medium 1 smaller than the optical density of
Which path of light ray is correct when the light medium 2.
ray enter the water?

Which path A,  B, C  or  D shows the correct


propagation of light ray O after through point
N?

6 Diagram shows a refraction of light when the


light propagates from air to perspex block.

3 Diagram shows ray of light propagate from


glass to air.

8
Which path of light ray is correct when perspex
block is replaced with a higher optically denser When the light travel from one medium to
block. another , in which case the change in speed will
be minimum and maximum?
Minimum change Maximum
of speed change of speed
A X to Y Y to X
B W to Z Z to W
C W to Z X to Y
D X to Y W to Z

11 Diagram shows a light ray travelling from


medium X to medium Y.
7 Diagram (a) shows a ray of light propagates
from medium  F to medium G .
Diagram (b)  shows a ray of light propagates
from medium  F to medium H .

 
Which of the following is true?

A The speed of light in medium X is larger


Arrange the optical density of the mediums, in than the speed of light in medium Y
ascending order. B The optical density of medium X is larger
than the optical density of medium Y
A G ,  F , H B H ,  F , G C The refractive index of medium X is larger
C F ,  G , H D F ,  H , G than the refractive index of medium Y  
D The angle of light to the normal in medium
8 Refractive index is defined as X larger than in medium Y.

frequency of light in vacuum 12 Diagram shows a ray of light directed to a


A
frequency of light in medium perspex block. 
frequency of light in medium
B
frequency of light in vacuum
speed of light in vacuum
C
speed of light in medium
speed of light in medium
D
speed of light in vacuum

Which statement is correct?


9 Which of the following comparison is true
A The light refracts away from normal as it
relates to a medium ?
enters the perspex block
Refractive Speed Size of
B The incident angle is equal to the refracted
index of light bending
angle
of light
C The light travels slower as it enters the
A Small Small Small perspex block
B Small Large Large D The brightness of light increases as it 
C Large Large Small travels in the perspex block
D Large Small Large
13 Diagram  shows a boy observing a stone in a
10 The refractive indices of four media W,X,Y pond. The stone appears to be closer to the
and Z are given by the following table surface of the water.

Medium W X Y Z
Refractive index ,n 1.33 1.50 1.52 2.40

9
Based on the information in the table, which
light ray, A, B, C or D, in diagram is correct?

Which reason is correct to explain this


situation?

A The refractive index of air > the refractive  16 Which path A, B, C or D, shows the correct
index of water propagation of light propagates from a medium
B The speed of light in air > the speed of  has small refractive index to a medium has a
light in water  large refractive index.
C The optical density of air > the optical
density of water
D The frequency of light in air > the 
frequency of light in water

14 Diagram shows a light ray  travels from liquid


P to liquid Q. 

Which of the following diagram correctly shows 17 A light ray passes through a window.
the path of the light ? Which path does it take?
[  Refractive index of liquid P > Refractive
index of liquid Q ] 

18 A light propagates from medium L into


medium M.
                       
15 Table shows the refractive indices of parafin
and ice.
Substance Refractive index, n
Parafin 1.48
Ice 1.31

10
Which of the following is a pair of angle of C 1.36 D 2.24
incidence and angle of refraction.
Angle of incidence Angle of refraction 23 Diagram shows a ray of light entering a
A h f transparent glass block.
B g e
C e g
D f h
19 The diagram shows a light ray which travels
from the glass to the air.

What is the refractive index of the glass block?


 
A 0.56 B   0.64 
C 1.54 D  1.78

What is the refrective index of the glass?  24 Diagram shows a beam of light travelling from
air into water.
sin w sin y
A n B n
sin y sin w

sin x sin z
C n D n
sin z sin x

20 Diagram shows a ray of light propagates from


medium 2 into medium 1. What is the rcfractive index of water?

A 1.14 B  1.19 
C   1.33 D  1.80

25 Diagram shows a light ray travels from the


air  into medium X.

Based on diagram which of the following is true


related to Snell’s law
What is the refractive index of medium X?
A n1sin c = n2 sin a
A 0.85 B 1.24
B n1sin b = n2 sin d
C 1.31 D 1.41
C n1sin a = n2 sin c
D n1sin d = n2 sin b
26 Diagram shows a light of ray  travels from
the air into a glass block. 
21 The speed of light in air is 3.0 x 108 m s-1. What
is the speed of light in olive oil of refractive
index 1.46?

A     4.8 x 1010 m s-1             B    4.5 x 109 m s-1 


C     4.4 x 108 m s-1               D   2.1 x 10s m s-1

22 The speed of light in the air is 3.0 x 108 ms-1 .


When the light move into a plastic block the
speed of light is 2.2 x 108 ms-1 .What is the
refractive index of the plastic block. What is the refractive index of the glass block?
A 1.38 B 1.45
A 0.73 B 1.33 C 1.51 D 1.62

11
27 Diagram shows a light ray propagating from
water to air. The refractive index of water is
1.33.

What is the angle s? 


At which point, A, B, C or D, is the image
A 22.1° B   41.7° position of the pencil?
C 48.3° D   60.0°
28 A beam of light going through a medium with 31 A man sees a fish in a lake.Which labelled path
refractive index of 1.14 has an angle of is taken by the light travelling from the fish to
incidence of 42∘ with another medium. When the man’s eye?
the light ray is at a new angle of 36∘, what is the
refractive index for the second medium?

A 0.86 B 1.00
C 1.17 D 1.30

29 Diagram shows a ray of light is incident through


a diamomd with refractive index 2.42, on an
interface separating diamond and parafin with
refractive index 1.48.
32 Diagram shows a coin is placed at the base of
a beaker. The image of the coin appears as
shown in the diagram.

What is the angle of refraction , g , the angle of


incidence of the ray in diamond is 30 °?
What is the refractive index of the liquid?
o o
A 54.8 B 52.3
C 37.7o D 35.2o 9
A B
4
30 Diagram shows a pencil in a beaker filled with 13
water. 9
20 16
C D
16 13
33 Diagram  shows an observer looking the iamge
of a fish swimming in a pond.

12
[ 2 marks ]
35 Diagram 35 shows a ray of light passes from air
to crown glass and then into water .

.
What is the distance bwtween the fish and the
observer’s eye.
4
[The refractive index of the water is ]
3
A   50.0 cm B   41.3 cm
C   20.0 cm D    11.3 cm

34 Diagram 34 shows a decorative lamp has a


transparent liquid in the space above a bulb.
Light from the bulb passes through rotating
coloured filters giving the different colours in
the liquid.
Diagram 35

Based on Diagram 35,

(a) Complete the ray diagram until the ray of


light emerges in air again.
[ 2 marks ]

(b) Calculte all angles between normals and the


boundaries of medium.

[ 3 marks ]
36 Diagram 36.1 and Diagram 36.2 show a ray of
Diagram 34 light with angle of incidence 70o propagates
(a) Name the light phenomenon involved at the from  air to block R and to block S respectively.
liqiud-air boundary.
........................................................................
[ 1 marks]
(b) Explain how the phenomenon occured.
........................................................................
........................................................................
........................................................................
........................................................................
[ 2 marks ] Diagram 36.1
(b) Calculate the refractive index of the liquid.

[ 2 marks ]
(c) The speed of light in air is 3 x108 m s-1.
Calculte the speed of the blue light in
the liquid.

13
Diagram 36.2
(a) Based on Diagram 36.1 and Diagram 36.2
compare
(i) the refractive index of the blocks.
..................................................................
[ 1 mark ]
(ii) the speed of light , v in the blocks. Diagram 37
..................................................................
[ 1 mark ] (a) Name the phenomenon of light involved
(iii)the angle of refraction in the blocks shown in Diagram 37.
.................................................................. …………………………………….…..........
[ 1 mark ] [ 1 mark ]
(b) Based on your answer in (a), states the (b) Underline the correct answer in the bracket
relationship between to complete the sentence below.
(i) the refractive index and the speed of light     Phenomenon in (a) happens because of   
in medium.            the change in (velocity, frequency) of the 
..................................................................     light.
[ 1 mark ] [ 1 mark ]
(ii) the speed of light and the angle of
refraction in medium. (c) Give one reason why the spear fails to hit
.................................................................. the fish.
[ 1 mark ] ..................................................................
[ 1 mark] 
(c) Diagram 36.3 shows a spoon is immersed in (d) Give one suggestion how to hit the
a beaker of water. fish accurately.
..................................................................
[ 1 mark] 

38 (a) Diagram 38.1 and Diagram 38.2 show


the positions of the image seen by the
observer when two identical coins are
placed in beaker containing with
Diagram 36.3 different depth of water.
On Diagram 36.3, complete the ray diagram
to show how the image of the spoon is seen.
[ 2 marks ]
37 Diagram 37 shows a man throws a spear
toward a fish but the spear fails to hit the fish.
.

Diagram 38.1

14
39 Diagram 39.1 and Diagram 39.2 shows two
identical aquariums but with different level of
water.
When a fish in the aquarium is observed from
the same position, the position of the images
of the fish are shown in the diagrams.

Diagram 38.2

(i) What is meant by by apparent depth.


[ 1 mark] 
(ii) Observe Diagram 38.1 and Diagram
38.2, compare the real depth and the
apparent depth of the coin and the Diagram 39.1
refractive index of the water.
State the relationship between real depth
and the apparent depth. Name the light
phenomenon involved.
[ 5 marks]
(b) Why do Stars twinkle? Explain your answer.
[ 4 marks ]
(c) Diagram 38.3 shows a ray of light AO
moves in a liquid which has refractive index
1.22 and then passes through a plastic
prism. OB is the path of the light ray in the
plastic prism.

Diagram 39.2
Based on your observations on the level of the
water and the position of the images of the fish:
  (a) State one suitable inference. 
[ 1 mark]
(b) State one suitable hypothesis. 
[ 1 mark]
(c) With the use of apparatus such as a tall
beaker, pins and other apparatus, describe an
experiment to investigate the hypothesis
stated in (b).
Diagram 38.3 In your description, state clearly the
following:
(i) Calculate the refractive index of the plastic (i) The aim of the experiment.
prism. (ii) The variables in the experiment..
[ 2 marks] (iii) The list of apparatus and materials.
(iv) The arrangement of the apparatus.
(ii) Draw the ray of light started from B to show   (v) The procedure used in the
the path of ray in the liquid again. experiment.Describe how to control and
Calculate the angle of refraction at B. measure the manipulated variables and
how to measure the responding
variables.
  (vi) The way to tabulate the data.
(vii)The way to analyse the data.
[ 3 marks]  [10 marks] 

15
At this moment , the total internal reflection
occured. The boundary of two mediums act as a
reflector.

2.1.2 Experiment to show how the total


internal reflection and to determine the
critical angle of the semi-circular glass block.

Aim of the experiment : 

To show how the total internal reflection and to


determine the critical angle of the semi-circular
glass block.

List of apparatus and materials:

Semi-circular glass block, ray box, white paper 


protractor and power supply 

LESSON 2 - TOTAL INTERNAL


Arrangement of the apparatus:
REFLECTION
2.1. How the total internal reflection occur?

2.1.1. Introduction.

As we already know , when the ray of light


propagates from large optical density medium to
the small density medium , refraction away
from the normal occured.

The procedure of the experiment


Place a semi-circular glass block on white
paper.
Draw the border of the semi-circular glass block
on white paper and mark as PQR.
But what happens when the angle of incidence Remove the glass block and mark the center of
is gradually increased. The angle of refraction the glass block as O
become larger until the ray of light back into the Draw normal ON.
large optical density medium. Place again the glass block back on the white
paper.

16
Turn on the ray of light and point the ray to
center O at an angle of incidence, i.
Repeat the experiment by adding the angle, i.
Observed what happens.
By using a protractor , measure the angle of
incidence when the angle of refraction is 90o,
=c

Observation
When the angle incidence is small , the
refraction and the reflection occur.
When the angle of incidence , i greater the angle
c , all ray of are reflected.

2.1.3 Ray diagram to show how the Total


Internal Reflection happen ?

Solution
(1) Angle of incidence ,i1 is small.
Produces a strong refracted ray and a weak
reflected ray.
(2) Angle of incidence is increased as i2.
Produces a  refracted ray and a reflected ray
whose intensity has increased compared to the
situation in (1)
(3) Angle of incidence is increased to the
critical angle, c.
The refracted ray travels along the water-air
boundary.
(4) Angle of incidence , i3  > c.
No refraction occurs.
All the light is reflected within the water .
Total internal reflection occurs
2.1.4 The conditions for the occurrence of
total internal reflection.

(1) The light ray must be travel from an


optically denser medium to less dense
medium. 2.2.2 Determine the relationship between
(2) The angle of incidence must be greater than Refractive index (n) and Critical angle (c) 
the critical angle.

2.2 The Critical Angle, c

2.2.1 The meaning of the critical angle

The angle of incidence when the angle of


refraction is 90o.

Example 1
Mark in the following diagrams the critical
angle with ”c” .

17
sin i
n
sin r
sin 90 o

sin c
1

sin c
As conclusion :
1
n
sin c
Example 2

The critcal angle of diamond is 24.4o. The procedure of the experiment which
Determine the refractive index of diamond. include the method of controlling the
manipulated variable and the method of
Solution measuring the responding variable.

Place a semi-circular ice block of refractive


index . n = 1.31on white paper.
Example 3 Draw the border of the semi-circular ice block
on white paper and mark as PQR.
The refractive index a glass is 1.51. Calculate Remove the semi-circular ice block and mark
the critical angle of the glass. the center of the block as O
Draw normal ON.
Solution Place again the semi-circular ice block on the
1 white paper.
n Turn on the laser pen and point the ray to center
sin c
O.
 1 
c  sin 1    41.47
o Increase the angle of incidence until the ray of
 1.51  laser light on OQ.
2.2.3 Experiment to investigate the By using a protractor , measure the angle of
relationship between the refractive index, n incidence = c
and the critical angle,c Experiment is repeated 4 times by using
different type of semi-circular block such as
Hypothesis:  Pyrex (n =1.47) , Perspex (n =1.49) , Crown
glass ( n = 1.52) and Flint glass ( n= 1.61)
As the refractive index increases , the critical
angle decreases. Tabulate the data:
Refractive 1.31 1.47 1.49 1.52 1.61
Aim of the experiment :  index, n
Critical
To investigate the relationship between the angle (o)
refractive index, n and the critical angle,c
Analysis the data:
Variables in the experiment:
 Plot the graph c against n
Manipulated variable: Refractive index
Responding variable: Critical angle
Constant variable: Diameter of semi-circular
block and colour of light

List of apparatus and materials:


Semi-circular block with different materials,
laser pen , white paper  and protractor
2.2.4 Experiment to show how the total
internal reflection in liquid occured
Arrangement of the apparatus:
Aim of the experiment

18
To show how the total internal reflection in
liquid occured.

List of apparatus and materials

Laser pen , binder clip , books , plastic drinking


bottle , cellophane tape, plastic basin , water
and cooking oil

Arrangement of the apparatus:

During hot day , the incidence ray from the sun


passes through a high optical density air to
small optical density air.
The ray of passes through the air layer close to
hot surface at an angle of incidence greater than
the critical angle of cool air.
Total internal reflection occurs at this layer.
The procedure of the experiment The observer sees the image of the sky on the
surface of hot layer as a pool of water.
Drill a hole near the bottom of the wall of bottle
with a pin has a diameter which is slightly 2.3.2 Formation of rainbow
larger than the diameter of the laser that will be
used.
Tape the hole and then fill the bottle with water.
Stand the soda bottle on top of a stack of books
so the hole is facing the plastic basin.
Carefully remove the tape and then unscrew the
top of the soda bottle and turn on the laser pen.
Obeserve the path of light in the stream of
water.
Repeat the experiment by using cooking oil.

Observation

The total internal reflection occur many times in


the stream of liquids until out from the end of When the white light from the sun enters the
the liquids stream. raindrop, it experiences refraction and
The number of total internal reflection in dispersion in seven different colors due to
cooking oil larger than in the water. different wavelengths(speed).
All diffrent of colors of light experience total
internal reflection at the surface of the raindrop,
After reflection they experince dispersion again
and the observer can see the rainbow.
Discussion 2.3.3 Sparkling of diamond
The number of total internal reflection in
cooking oil larger than in the water because the
cooking oil has the refractive index larger than
water. So that the critical angle of the cooking
oil smaller than the water caused the total
internal relection easy to occur.

2.3 Natural phenomena and Use of Total


Internal Reflection.

2.3.1 Formation of mirages

19
Diagram shows a structure of a periscope.

The optical density of diamond is high


and caused the the refractive index is
large.
As the result the critical angle of
diamond is small ≈24.4o
When the light ray enters the diamond ,it
experiences multiple total internal
reflections inside the diamond.
 After many such reflections, the colors Complete the ray diagram to shows how the
in the light are separated, and seen image the object formed?
individually.

2.3.4 Prism periscope


Solution

The periscope is built using two glass


prisms with its angle is 45o-90o- 45o
The critical angle of the glass is 42o .
The angle of incidence is 45o is greater than
the critical angle.
Total internal reflection occurs.   

The characteristics of the images are


(i) Virtual
(ii) Upright
(iii) Same size as the object.

Advantages  of the prism periscope


compared to mirror periscope,
(i) The image is brighter
(ii) No multiple images is formed (very
clear)

Example 4
2.3.5 Binocular

20
A ray light experiences two times of the
total internal reflections at each prism which
have the angles are 45o-90o- 45o .The two
prisms are to invert the image (upside down
and right-to-left.) But the lenses in the
binoculars also invert the image and so the
prisms put it back the right way again.
So the final image in binoculars is upright
,not laterally inverted and magnified.
Bundle of optical fibers
Example 5
An optical fiber is a flexible ,tranparent fiber
made by glass or plastic to a diameter
Diagram shows the arrangement of prisms
slightly thicker than that of a humam hair.
in a binocular.
The optical fibers are assambled in a bundle
to split or to combine signals.
The external outer cladding has the a small
optical density compare to the inner core.
When light rays travel from the inner core
to the outer cladding at an angle that exceeds
the critical angle, total internal reflection
occurs until the ray out at the end of the
optical fibre.
The advantages of the fibre optics:
(i) Cheap
(ii) Easily handled 
(iii)More information can be transmitted and
received.
Complete ray diagram in the prisms.

Solution 2.3.7 Reflector on the road and bicycle


reflector

Reflector on the road (Cat’s eye)

Bicycle reflector
  2.3.6 Optical fiber

21
(ii)

(iii)

Example 7
The reflector is made up of many small
persepex prisms arranged so that light Diagram shows a rectangular glass block PQRS. A
undergoes total internal reflection twice. ray of light MO is directed to the side PS and then
The overall result is that the light is returned into the glass block and reach to the side PQ at
in the direction from which it origanally point N. The refractive index of the glass is 1.51.
came.

2. 4 To solve some problem involving the total


internal reflection

Example 6

Diagram shows a light ray AO travelling from


medium X  to the air.
[The refractive index of medium X = 2.0 ]
(a) Calculate the critical angle of the glass
block.
(b) By determining the certain angles on the
glass blocks complete the light path starting
from N.

Solution
(a)

(a) Calculate the crtical angle of the meduim X.


(b)
(b) Draw the ray light after it passes through the
point  O when 
(i) θ = 25o     (ii)θ = 30o (iii) θ = 35o 

Solution

(a)

(b) (i)

22
Example 8 TUTORIAL 2
Diagram shows a light ray entering in a glass prism.
1 A ray of red light travelling in glass strikes the
[ Refractive index of glass = 1.51 ]
glass-air boundary . Some light is reflected and
some is refracted. Which diagram shows the
paths of the rays?

(a) Determine the critical angle of the glass.


(b) Draw the ray light after it passes through a point
P until the direction of the ray of light
unchanged.
   
Solution

(a)
   
(b)                          
2 Which of the following diagram shows the
phenomenon of total internal reflection?

3 Which of the following diagram shows the


path of a beam of light that is incident on a
water-air surface with angle of incidence greater
than the critical angle.

23
8 A ray of light incident on one side of a
4 Which of the following diagram correctly shows rectangular plastic block. When the angle of
the total internal reflection of ray of light?  refraction in the plastic block  is 42o ,which one
[ The refractive index of medium X > Y)  of the following diagrams best represents this
ray?
[ The critical angle of glass is 44o ]

5 The critical angle of a type of coloured liquid is


47°.Which diagram shows the total internal
reflection?

         

9 Diagram shows a ray of light starting from M


entering a vertically angled glass prism and has
an angle of 42o.
6 The figure shows a ray of light PO  traveling in After passing through the point O,Which of the
a liquid strikes the liquid-air boundary.  trajectories of A, B, C or D is the actual
[ The critical angle of the liquid = 45o ] trajectory of the beam of light?

In which direction does the light move from O ?

A OQ B OR 10 A ray of light incident on one side of a glass


C OS D OT prism of angle 45o – 90o – 45o at a right angle
to the side.
7 Diagram shows a ray of light in a semi-circular Which one of the following diagrams best
glass block. represents this ray?
If the critical angle of the glass is 42o, what is [ The critical angle of glass is 42.2o ]
the correct path for the light?

24
11 Which diagram shows a ray hitting asemi- 15 Diagram (a) and Diagram (b) show light rays
circular glass block at the critical angle, θ?  travelling from perspex to air at different
incident angles.

Diagram(a) Diagram(b)

Which angle is the critical angle of the glass? 


12 Diagram shows the path of light from P to O.  
A p B q
C r D    s

16 Which of the following shows the correct


critical  angle , c of the semi- circular glass
block

The critical angle of water is 48°. When


θ° = 48°, which is the correct path of light after
passing through point O?

A OR B   OS
C OT D   OU

13 Diagram shows a light ray X is directed into the


glass block. The critical angle of the glass is
42 0o. Which direction does the ray X propagate
from point Y?     
17 Diagram shows a light  ray travelling from air
into a plastic block with an angle of incidence
,X. What is the critical angle of the plastic?

14 Which of the path of ray formed the critical


angle of water?
18 Diagram shows a light ray travelling from air
into a prism X.

25
What is the critical angle of the prism X?

A 40o B 50o
C 60o D 70o

19 The refractive index of parafin is 1.48


What is the critical angle of the parafin.

  A 28.7o B 34.9o
  C 42.5o D 47.5o 

20 Diagram shows a ray of light propagates in a In which direction does the light move from O ?
transparent semi -circular block.
A OE B OD
C OC D OB

23 The diagram shows  a semi-circular plastic


block is placed  in a liquid.

What is the possible value of refractive index of


the transparent block? 

A 1.55 B   1.63
C 1.73 D   1.81
Which of the following is correct?
21 Diagram shows a ray of light passing through a
prism with refractive index of 2.00 A The optical density of  the plastic block is
less than the optical density of the liquid
B The refractive index of  the plastic block is
less than the refractive index of the liquid
C The the critical angle of  the plastic block is
less than the critical of the liquid
D The angle of incidence in plastic block is
less than the critical angle of the liquid
Which of the following diagram correctly shows
the trajectory of an incident ray through a prism? 24 Diagram shows a ray of light  is incident in air
to the surface of Prism A  and B.

Which comparison is correct ?

Prism A Prism B
A Small optical Large optical
density density
B Small critical Large critical
angle angle
22 Diagram shows a ray of light AO  traveling in C Small refractive Large refractive
strikes the medium X-air boundary.  index index
[ The refractive index of medium X = 1.12 ] D Small angle of Large angle of
incidence incidence

26
25 Diagram shows a ray of light passing through 29 Which of the following not applies the principle
medium M to medium N. of total internal reflection? 

A Reflector on the road


B Mirror  periscope
C Prism binocular
D Road mirage
30 Diagram shows a binocular containing prisms.

  
Which of the following is correct?

A The angle of reflection is 55o Which arrangement of prisms is correct so that


B The critical angle of medium M less than an observer can see the image of the object?
35o
C The optical density of medium  M less than
the optical density of medium N
D The refractive index of medium  M less
than the refractive index of medium N

26 Which  of the following shows total internal


reflection?

31 Rainbow is seen on sky after rain


Which phenomena cause the appearance of
rainbow?

27 Which instrument uses the concept of total A Refraction  


internal reflection? B Refraction and total internal reflection 
      CTotal internal reflection and dispersion
  A Microscope  D Rrefraction , total internal reflection and
B Magfying glass  dispersion
C Prism periscope
D Astronomical telescope  32 Diagram 32 shows light beam strikes the edge
of a rectangular block of Perspex WXYZ of
28 Which of the following diagram not involving refractive index 1.51
total internal reflection?

27
Diagram 32 (i) Draw two straight lines from the symbol
(a) Calculate the magnitude angle of of angles to the suitable name of
(i) m angles.

Symbol of angle Nane of angle

incidence angle
i
refraction angle
[2 marks]  c
critical angle
(ii) n [ 1 mark ]
(ii) Compare the refractive index of the
materials.
[1 mark]  ..................................................................
[ 1 mark ]
(b) Calculate the critical angle of the perspex (b) Based on Diagram 33.1 and Diagram 33.3 ,
block. compare
(i) the size of bending of the light in the
materials.
..................................................................
[2 marks]  [ 1 mark ]
(c) On Diagram 32, starting from P complete (ii) the optical density of the materials.
the ray diagram until the ray emerges in air ..................................................................
again. [ 1 mark ]
[2 marks]  (c) Based on Diagram 33.2 and Diagram 33.4,
compare the angle of ‘c ‘of the materials.
33 Diagram 33.1 and Diagram 33.2 show material ........................................................................
X with a refractive index ,n of 1.32 cut into [ 1 mark ]
rectangular block and semi-circular block (d) Based on your answers in (a), (b) and (c) ,
respectively. state the relationship between
Diagram 33.3 and Diagram 33.4 show material (i) the refrcative index and the optical
Ywith a refractive index , n of 1.46 cut into density
rectangular block and semi-circular block ..................................................................
respectively. [ 1 mark ]
The ray of light from air is directed to the (ii) the optical density and the angle of ‘c’.
blocks. ........................................................................
O is the centre of the semi-circular blocks. [ 1 mark ]
(e) Explain why the semicircular blocks are
used.
.......................................................................
........................................................................
[ 2 marks ]
34 Diagram 34.1 shows a man watches whales
Diagram 33.1 Diagram 33.2 in an ocean by using a binocular. Diagram
34 2 shows the strucutre of one side ofthe
binocular.

Diagram 33.3 Diagram 33.4

(a) Based on information in Diagram 33.1 ,


Diagram 33.2,Diagram 33.3 and Diagram
33.4 :

Diagram 34.1 Diagram 34.2

28
(a) On Diagram 35.2 ,
(a) State one characteristic of the image (i) Mark the right angle in the prism.
produced by the binocular. [ 1 mark ]
........................................................................ (ii) Complete the path of the light ray from
[ 1 mark ] car headlamp to the driver.
(b) On Diagram 34.3, [ 2 marks ]
(i) draw the correct position of the prisms in (b) Why the light ray does not bend when it
the box P and Q. enters the prism.
[ 1 mark ] ………………………………………............
(ii) complete the path of the light ray from ……………………………………................
the object to the observer. [ 1 mark ]
[ 2 marks ]  (c)(i) Name the phenomenon involved.
(c) You are required to  modify nthe binocular ………………………………………......
that can produce sharper and clearer image [ 1 mark ]
as well as more portable during the (ii) Why the phonomenon in (c)(i) occurred?
whales. ..................................................................
State and explain the modificattions based ..................................................................
on the following aspect [ 2 marks ]
(d) The refrcative index of perspex is 1.50.
(i) Focal length of the objective lens Calculate the critical angle of the perspex.
..................................................................
Reason: 
.................................................................. [ 2 marks ]
[2 marks ] 36 (a) Diagram 36.1 shows prism M and prism N
(ii) Diameter of the lens: has the critical angle 42o and 47o
.................................................................. respectively.
Reason: 
...............................................................
[2 marks ]
(iii) The characteristics of material used for
the body of the binocular:
..................................................................
Reason: 
..................................................................
[2 marks ] Diagram 36.1
(d) The binocular and a periscope work (i) What is meant by critical angle?
based on the phenomenon of total [ 1 mark ]
internal reflectionb but what’s different? (ii) Based on Diagram 36.1 , compare the
........................................................................ angle of incidence at P , comparison the
[ 1 mark ] magnitude the angle of incidence with
35 Diagram 35.1 shows a reflector on a road critical angle and the light phenomenon
known as cat’s eye fixed into a road to help occur after point P.
drivers when it is dark or foggy . Relate the angles involved with the light
Diagram 35.2 shows the structure of the cat’s phenomenon occur.
eye containing perspex prism. [ 5 marks ]
(b) Diagram 36.2 shows an observer looking 
image of a sailboat at sea during night time.
The image of the sailboat appears in inverted
position in the sky.
Diagram 35.1

Diagram 35.2

29
Diagram 36.2

Explain that observation 


  [4 marks] Table 39 shows the design and the
(c) Diagram 36.3 shows a light signal travelling specifications of four periscope  P, Q, R and
through an optical fiber made of glass. S used glass prisms and lenses.Study the
specifications of all four periscope. Explain
the suitability of each design and its
specifications. Determine the most suitable
periscope to be used by the observer. Give
reasons for your choice.
[ Critical angle of glass =  41.8o ]

Periscope P Q R S

Angles of 45o , 90o ,  30o , 45o , 90o ,  30o ,


prism 45 o
60 ,  90 45o
o o
60o ,  90o
Diagram 36.3
  The optical fibre in Diagram 36.3.can be 
used in telecommunications and medicine. Arrangem Type I Type II Type II Type I
Give your suggestions to make the optical fiber ent of
more efficient based on the following aspects: prisms
-Comparison between refractive index of the
inner core, and the outer cladding
-The flexibility
Type of Con Con Convex Convex
- Material of the jacket
lenses cave cave
- Purity of inner core
- Assemble in single fiber or in bundle fibre
[ 10 marks ] Shape Type I Type II Type I Type II
37 (a) Diagram 37.1 shows a fish is behind a large of outer
rock in a pond while a shrimp were on the casing
other side of the stone.
Table 37
[10 marks] 

(c) Diagram 37.3 shows  a ray of light AO is


incident to a glass prism UVW without
change its direction until reach at B.
[The refractive index  of glass = 1.5] 

Diagram 37.1
(i) State one phenomenon of light involved
 [1 mark]
(ii) Explain how the fish can see the shrimp
without going to the other side of the
stone.
[4 marks]
(b) Diagram 37.2 shows an observer use a
periscope to see a distant object behind a
high wall.

Diagram 37.3

Calculate the critical angle of the glass and


hence draw the complete path of a ray started
Diagram 37.2

30
from B until the direction of the path of ray
unchanged.  
 [5 marks]

38 Diagram 38.1 and Diagram 38.2 show two LESSON 3 – FORMATION OF


identical adjustable sources of light are directed IMAGE BY LENSES
two surfaces of liquid with different optical
densities respectively. The sources of light are
adjusted until the rays of light emerge exactly
parallel to the surface of the water. 3.1.Introduction

A lens is a transparent material which consists of at


least one curved surface.
For example , the surface of water waves can acts
as convex lens(crest) and concave lens (trough) ,
the glass stem of thermometer in curved-shaped
acts as convex lens and adrinking transparent bottle
filled with liquid acts as a convex lens.
Images formed by lenses due to the phenomenon of
Diagram 38.1 refraction of light.
Rays of light from objects at infinity ( distant 
objects) are parallel rays. 
There are two types of lens i.e.convex lens and
concave lens.

Diagram 38.2
Based on your observations on the level of the
water and the position of the images of the fish:
  (a) State one suitable inference. 
[ 1 mark]
(b) State one suitable hypothesis. 
[ 1 mark]
(c) With the use of apparatus such as
laser pointer, protractor , white paper
and other apparatus, describe an
experiment to investigate the hypothesis
stated in (b).
In your description, state clearly the
following:
(i) The aim of the experiment.
(ii) The variables in the experiment.. 3.2 To identify a convex lens as a converging lens
(iii) The list of apparatus and materials. and concave lens as a diverging lens.
(iv) The arrangement of the apparatus.
  (v) The procedure used in the 3.2.1 Experiment to show a convex lens as a
experiment.Describe how to control and converging lens and concave lens as a
measure the manipulated variables and diverging lens.
how to measure the responding
variables. Aim of the experiment
  (vi) The way to tabulate the data.
(vii)The way to analyse the data. To show how a convex lens as a converging
 [10 marks]  lens and concave lens as a diverging lens.

List of apparatus and materials

31
Ray box, convex lens, concave lens,plate with
parallel slits , power supply, white paper and
pencil.

3.2.2 Common terminology of lenses

Arrangement of the apparatus:

The procedure of the experiment Optical centre , O


  
Place a convex lens in front of the ray box fitted A point which all rays traveling through this
with a multi-slit plate, onto a sheet of a white point pass through the lens in a straight line. 
paper.
Trace around the lens with a pencil. Principal axis  , AB
Switch on the bulb in the ray box on and use a
pencil to to show the path of the rays of light A straight line passing through the optical
before and after passing through the lens. centre of a lens and joining the two centres 
Experiment is repeated by using a concave lens. of curvature of its surfaces and at a right angle
to the plane of the lens.
Observation
Focal point ,F

For convex lens : A point behind the lens and


on the principal axis in which the parallel rays
of parallel to the principal axis are converged
after passing through the convex lens.

Discussion For concave lens : A point in front the lens and


on the principal axis in which the parallel rays
For the convex lens the parallel of light is of parallel to the principal axis are started to
converged to a point behind the lens. The point diverged before passing through the concave
known as the focal point and the point is real. lens.
The focal length of the convex lens is positive.
Focal length , f
For the concave lens the parallel of light is
diverged from a point in frontof the lens. The The distance between the optical centre ,) and
point known as the focal point and the point is the focal point,F.
virtual.The focal length of the convex lens is
negative. Object distance , u

Distance of the object from the optical centre,O

32
Image distance , v image of concave lens always virtual (Cannot
form on the screen)
Distance of the image  from the optical centre,O

3.2.3Experiment to estimate the focal length


of a convex lens.  3.2.4 Relationship between the thickness and
the focal length of the lens
Aim of the experiment

To estimate the focal length of a convex lens by


observing the real image of a distant object. 
.
List of apparatus and materials

Convex lens, lens holder , white screen and


ruler.

Arrangement of the apparatus:

Based on diagram above , the conclusion is


the thicker the lens ,the shorter the focal length.

3.2.5Experiment to investigate the


relationship between the thickness lens and
the focal length for a convex lens. 

Hypothesis: 

As the thickness of lens increases , the focal


length decreases
The procedure of the experiment
Aim of the experiment : 
A convex lens is turned to face a distant object
located outside the laboratory To investigate the relationship between the
The position of the screen is adjusted until a thickness and the focal length for a convex
sharp inverted image is formed on the screen. lens. 
The distance of the screen from the optical
centre of the lens is measured by using ruler = Variables of the experiment:
x1
The experiment is repeated 4 times by facing Manipulated variable: thickness of lens
the lens with differnt distant objects. Responding variable:  focal length
Record the distance from screen and optical Constant variable: diameter of  lens, refractive
centre of the lens as x2 , x3 , x4 and x5 , index of  lens

Calculation List of apparatus and materials:

The focal length , f of the convex lens is


Screen ,convex lens,lens holder and metre ruler
calculated ,
Arrangement of the apparatus:
x1  x 2  x 3  x 4  x 5
f
5

Discussion

This experiment cannot be used to determine


the focal length of a concave lens ,because the

33
The procedure of the experiment which
include the method of controlling the
manipulated variable and the method of
measuring the responding variable. Ray 2

Measure the thickness of the centre of lens by


using metre ruler ,D = 3cm
The lens is faced to an object at infinity
outside the laboratory
The screen is adjusted until a sharp image
formed.
Measure the distance between the screen and
the lens = f
The experiment is repeated 4 times for D = 5 cm
, 8 cm, 11 cm and 14 cm.

Tabulate the data:

Tabulate the data


D 3 5 8 11 14
(cm ) Ray 3
f (cm)

Analysis the data:

 Plot the graph f against  D

3.3 Characteristics of image formed by lenses by


using ray diagrams.

3.3.1 Ray diagram of lenses The method to complete ray diagram for
convex lens and concave lens are:
The main ray of light in drawing a ray diagram
are: 1.Draw Ray 1 from the top of object.
2.Draw Ray 2 or Ray 3 from the top of the
Ray I object.
3.Extrapolate Ray1 and Ray 2 or Ray 3
until intersect each other.
4.Draw image at the intersection point.
34 5.Draw arrow from object to the image
Example 1

Complete the following ray diagram and state the Example 3


characteristics of image formed
(a) Complete the following ray diagrams for
convex lens
(b) State the characteristics of image formed
(c)Complete the table below

(a) u < f

Solution

(b) u = f
Example 2

Complete the following ray diagram and state the


characteristics of image formed

(c) u < f < 2f

Solution

35
The distance Characteristics of image
object, u
u < f
u =f
f <u < 2f
u = 2f
(d) u =2f
u > 2f
u=

Solution

(a) u < f

(e) u > 2f

Characteristics of image:
.............................................................................
.............................................................................
(b) u = f

(f) u = 

36
Characteristics of image: Characteristics of image:
............................................................................. .............................................................................
............................................................................. .............................................................................
(f) u = 
(c) u < f < 2f

Characteristics of image:
Characteristics of image: .............................................................................
............................................................................. .............................................................................
.............................................................................
(d) u =2f The distance Characteristics of image
object, u
u < f
u =f
f < u < 2f
u = 2f
u > 2f
u=
Example 4

(a) Complete the following ray diagrams for


concave lens
Characteristics of image: (b) State the characteristics of image formed
............................................................................. (c)Complete the table below
.............................................................................
(a) u < f
(e) u > 2f

37
Characteristics of image:
.............................................................................
(b) u < f < 2f ...........................................................................
Object distance ,u Characteristics of image
 u<f
f < u < 2f

3.4 Linear magnification ,m

3.4.1 The meaning of linear magnification ,m

Solution The linear magnification , m of image for by a


lens is given by ;

(a) u < f
height of image
Linear magnification 
height of object
h
m i
ho
Also

The linear magnification , m of image for by a


lens is given by ;

Image distance
Linear magnification 
Characteristics of image: Object distance
............................................................................. v
............................................................................. m
u
(b) u < f < 2f
Linear Size  of image
magnification,
m
I m I  = 1 Image  and object are the
same size
I m I  > 1 Enlarged(magnified)
image
I m I  < 1 Diminished image

38
3.4.2 Experiment to determine the linear Aim of the experiment :
magnification of image for an object formed
by a lens. To investigate the relationship between the
object distance ,u and the linear
Aim of the experiment magnification , m.
To determine the linear magnification of image Variables in the experiment:
for an object formed by a lens
Manipulated variable: object distance, u
List of apparatus and materials Responding variable: linear magnification ,
m.
Convex lens, lens holder , white screen and Constant variable: focal length of the lens, f
ruler, cardboard with a cross-wire in a triangular
cut-out(object) List of apparatus and materials:
Arrangement of the apparatus: Convex lens, lens holder , white screen and
ruler, cardboard with a cross-wire in a
triangular cut-out(object),

Arrangement of the apparatus:

The procedure of the experiment

The distance between the object and the


convex lens is measured by using a ruler ,
u1 = 20 cm The procedure of the experiment which
The bulb is lighted up include the method of controlling the
The screen is moved back and forth until a manipulated variable and the method of
sharp image is formed on it. measuring the responding variable.
The distance between the screen and the lens
is measured , v1. The distance between the object and the
The experiment is repeated 4 times for the convex lens is measured by using a ruler ,
other object distances u2 = 30 cm, u3 = 40 u = 20 cm
cm, u4 =50 cm and u5=60 cm The bulb is lighted up
The screen is moved back and forth until a
Calculation sharp image is formed on it.
The distance between the screen and the lens
The linear magnification is calculated , is measured , v.
v1 The linear magnification is calculated ,
m1  v
u1 m
Calculate the linear magnification , m2 , m3 , u
,m4 and , m5. The experiment is repeated 4 times for the
Calculate the average linear magnification , other object distances ,u= 30 cm , 40 cm., 50
m cm and 60 cm.
m1  m 2  m 3  m 4  5 Tabulate the data:
m
5 u(cm) 20 30 40 50 60
3.4.3 Experiment to investigate the m
relationship between the object distance
Analysis the data:
,u and the linear magnification , m.
Plot the graph m against u
Hypothesis:

As the object distance ,u increases ,the


linear magnification decreases.

39
3.5 Thin lens equation

3.5.1 Thin lens equation


The procedure of the experiment
Thin lens equation is given by:
1 1 1
  The distance between the object and the
f u v convex lens is measured by using a ruler ,
Where ;  f = focal length  u = 20 cm
u = object distance The bulb is lighted up
           v = image distance The screen is moved back and forth until a
                     sharp image is formed on it.
3.5.2 Sign Conventions for lens The distance between the screen and the lens
equation is measured , v.
The experiment is repeated 4 times for the
Convex lens Concave lens other object distances ,u= 30 cm , 40 cm., 50
Object + if object in front + if object in front cm and 60 cm.
distance , of lens and – if of lens and – if 1 1
u object behind the object behind the Plot the graph against
lens lens u v
Image + if image behind + if image behind 1 1
Determine the - intercept and -
distance , of lens (real of lens (real u v
v image) and – if image) and – if intercept
image in front the image in front the
lens(virtual image) lens(virtual image) Calculation
Focal + - 1
- intercept = a
length, f u
3.5.3 Graph of lenses based on the thin lens 1
equation - intercept = b
v
1 1
f1  and f 2 
a b
Calculate the average focal length
f1  f 2
f
2
3.4.5 Experiment to investigate the
3.5.4 Experiment to determine the focal relationship between the object distance
length of a convex lens by using the thin lens ,u and the image distance , v.
equation
Hypothesis:
Aim of the experiment
As the object distance ,u increases ,the
To determine the focal length of a convex image distance,v decreases.
lens by using the thin lens equation
Aim of the experiment :
List of apparatus and materials
Convex lens, lens holder , white screen and To investigate the relationship between the
ruler, cardboard with a cross-wire in a triangular object distance ,u and the image distance, v.
cut-out(object)
Arrangement of the apparatus: Variables in the experiment:

Manipulated variable: object distance, u


Responding variable:image distance,v
.

40
Constant variable: focal length of the lens, f (i) the image distance
(ii) the linear magnification
List of apparatus and materials: (iii)the image height
(b) State the characteristics of the image formed
Convex lens, lens holder , white screen and
ruler, cardboard with a cross-wire in a Solution
triangular cut-out(object),

Arrangement of apparatus
Example  6

An object of height  6 cm is placed at 20 cm 


froma concave lens of  focal length 40 cm.

(a) Calculate
(i) the image distance
(ii) the image height
(iii)the linear magnification
The procedure of the experiment which (b) State the characteristics of the image
include the method of controlling the formed
manipulated variable and the method of
measuring the responding variable. Solution

The distance between the object and the


convex lens is measured by using a ruler ,
u = 20 cm
The bulb is lighted up
The screen is moved back and forth until a Example 7
sharp image is formed on it.
The distance between the screen and the lens An object is placed 20 cm in front a lens. The
is measured , v. lens produced an image 4 times bigger than
The experiment is repeated 4 times for the objectalso located in front the lens.
other object distances ,u= 30 cm , 40 cm., 50 (a) Determine the focal length of the lens
cm and 60 cm. (b) What type of lens is used.

Tabulate the data:


Solution
u(cm) 20 30 40 50 60
v(cm)
Analysis the data: 3.6 Some important facts about lenses

Plot the graph v against u 1 A real image is one which can be formed on a
screen. while a virtual image is one which
cannot be formed on a screen.

2 When part of a lens is blocked by an object,


.  the size and position of the image is the same as
before. The brightness of the image, however ,
is reduced.
3.5 To solve some problem involving the thin
3 The larger the diameter of a lens , the larger the
lens equation
amount of light enter the the lens and as a result
the brighter the image formed.
Example  5
3.7 Optical devices use lenses
An object of height  2 cm is placed at 30 cm 
from a convex lens of focal length 20.0 cm.
3.7.1 Introduction
(a) Calculate

41
There are many types of optical devices used
lenses such as magnifying glass, microscope, Example 9
telescope, camera in smart phone or
CCTV,photocopier machine ,slide projector  Complete ray diagram for the telescope as
etc. shown below and state the characteristics of the
For every types of the devices we must learn final image.
about the uses of the instruments, lens
characteristics is used, normal adjustment of 
the instruments , ray diagrams and the
characteristics of the final image which are
formed .

3.7.2 Magnifying glass


Function To magnify image of an
object
Structure A thick convex lens
Normal Object distance, u  <
adjustment focal length , f
Ray diagram ?
Characteris tics ?
of final image

Example 8

Complete ray diagram for the magnifying as


shown below and state the characteristics of the
final image.

Solution:

Solution

3.7.3 Telescope 

Function To see a distant object


( object at infinity)
Structure Objective lens ( very long
focal length)
Eyepiece : (very short focal
length)
fo >>>> fe
Normal Fo and  Fe at same point 
adjustment or the distance between
two lenses =  fo  +  fe
Magnification of fo
telescope m
fe
Ray diagram ?
Characteris tics ?
of final image

42
3.7.4 Compound microscope

Function To see a tiny object


Structure Objective lens ( short focal
length)
Eyepiece : (short focal
length)
                  fe > fo
Normal Normal adjustment :  fo < u 
adjustment < 2fo

Ray diagram ?
Characteris tics ?
of final image

Example 10

Complete ray diagram for the compound


microscope as shown below and state the
characteristics of the final image.

43
Solution Example 11

Complete ray diagram for the camera as


shown below and state the characteristics of
the final image.

Solution

3.7.5 Small size lens in optical equipment


technology

Function As a main component of a high


resolution camera in smart
phone and CCTV
to capture and record pictures.

Structure Small size of convex lens. The


minimum distance between
optical centre of lens and sensor
= the focal length of the lens.
A sensor or mirror are used to
flip the inverted image to the
upright image
Normal Object distance u > 2f
adjustment
Ray ?
diagram
Characteris ?
tics of final
image

44
3.7.6 Convex lens as a paper burner TUTORIAL 3
1 The image produced by a lens is caused by  the
Function To burn small pieces of paper or
dried leaves
Structure A thick convex lens A total internal reflection of ray
B diffraction of ray
Normal Object distance, u = 
C refraction of ray
adjusment
D reflection of ray
Ray diagram ?
Characteristics ? 2 Which of the following acts as lens except
of final image
A The surface of water waves
Example 12 B Glass block in rectangular shaped
C The glass stem of thermometer in curved-
Complete the ray diagram of a convex lens using shaped
sunlight to burn the paper as shown in the diagram D The tranparent liquid in trasparent curved –wall
below and state the characteristics of final image in plastic drinking bottele

3 Which diagram is true when rays of light passing


through a liquid in a bottle.

Solution

4 Diagram shows a combination of two mediums


with different optical density.

Which diagram is true when the rays of light


passing trough the combination of mediums?

45
3 5 Which of the following drawing is not correct 
path of the light rays? 

10 Diagram shows ray diagram of a concave lens


in which F is the focal point.

6 Diagram show light rays passing through a


concave lens.

Which of the following equation is true?

OQ OP OQ OP
A  B 
IJ IP IP IJ
Which of the ray of light passing through the C OQ x IJ  OP x IP D
optical centre of the lens? OQ x OP  IJ x IP

A J B K 11 Diagram shows the ray diagram for a convex


C L D M lens.Which point is the focal point of the lens?

7 Diagram shows ray diagram of a convex lens.

12 Diagram shows the ray of light passing


through a concave lens.

Which part in the diagram represents image


distance?

A I B II
C III D IV

8 Diagram shows four light rays A,B,C and D


passing through a convex lens. F is the focal Which of the point A, B, C or D, is the
point of the lens. Which of the following path focal point of the lens?
of the light rays is not correct?
13 Which ray diagram is correct for a concave
lens? 

9 Which diagram shows the correct refraction of


light ray after passing through a lens?

46
16 Diagram shows an image ,I is formed by a 
concave lens. Where is the position of the
object?

17 Diagram shows the position of image formed by


a convex lens.

14 Which ray diagram is correct for a convex


At which position the observer can see the
lens? 
image?

A P and Q
B P and S
C R and S
D Q and R

18 A convex lens of focal length ,f.  The lens


produces a magnified , virtual  and upright
image. The object distance is

  A less than f
B between  f and 2f
  C same as 2f
D more than 2f

19 Diagram shows an object is placed in front of a


convex lens.

Based on diagram , what is the characteristics


of the image formed?

15 Diagram shows an image ,I is formed by a  A Magnified , inverted , real


convex lens. Where is the position of the object? B Diminished , upright , virtual
C Magnified , upright , virtual
D Diminished ,inverted ,real

20 Diagram shows an object placed in front of a


convex lens.

47
At which position is the focal points of the lens?
25 The image produced by a concave lens is,
A J and L
B J and M A enlarged, virtual, upright
C K and L B diminished, virtual, upright
D K and M C enlarged, real, inverted
D diminished, real, inverted
21 Diagram shows a bulb is placed in front a
convex lens at a distance same as the focal 26 Diagram shows an object is placed in front of a
length of the lens. concave lens at point P produced smaller ,
upright and virtual image.
F is the focal point of the lens.

After travelling through the lens the rays of light What is the characteristis of the image when
is the object is placed at point Q?

A parallel A bigger , upright , real


B diverged B bigger , inverted , virtual
C converged C smaller , inverted , real
D converged and diverged D smaller , upright , virtual

22 Diagram shows an object placed in front of a 27 Diagram shows a ray diagram of a convex
convex lens. lens.

What are the characteristics of the image 


formed?
What is the focal length of the lens?
A Real, inverted, diminished
B Virtual, inverted, same size A 15.0 cm B 24.0 cm
C Virtual, upright, enlarged  C 24.3 cm D 30.0 cm
D Real, inverted, enlarged
23 The height of image formed by a convex lens of 28 An object is placed at  12.0 cm from a concave
focal length , f has same height of the object. lens of focal length 20.0 cm.What is the image
The object distance, u is distance?

A u<f A 30.0 cm in front of the lens


B u=f B 30.0 cm behind the lens
C u = 2f C 7.5 cm in front of the lens
D u > 2f D 7.5 cm behind the lens

24 Diagram shows a convex lens of focal length f. 29 Diagram shows an image of an object is
At what point the object should be placed to formed by  a convex lens.
produce real , inverted and diminished image.

48
34 An object is placed 25 cm in front of a convex
lens and its image is formed at infinity. If the
object is placed 40 cm in front of the lens , the
image is

A inverted , magnified , virtual


B upright , diminished , real
What is the linear magnification of the image?  C inverted , magnified , real
D upright , diminished, virtual
A 0.5 B 1.0
C 3.0 D 4.0 35 A  convex lens of focal length 30.0 cm is turned
to face a distant object. The position of the
30 Diagram shows a ray diagram a convex lens. screen is adjusted until a sharp inverted image is
formed on the screen.What is the distance of the
screen from the lens ? 

A 15.0 cm B 30.0 cm
C 45.0 cm D 60.0 cm

36 The image of an object located 80.0 cm in front


for a concave lens with a focal length of 20.0
cm is
Based on diagram , what is the value of H?
A 16.0 cm at the same side of the object.
A 6.7 cm B 9.6 cm B 16.0 cm at the opposite side of the object.
C 15.0 cm D 21.6 cm C 26.7 cm at the same side of the object.
31 An object is placed at a distance of 30.0 cm D 26.7 cm at the opposite side of the object.
from a convex lens with a focal length of 25.0
cm.What is  the linear magnification ? 37 Diagramshows an object which is placed at
u cm from centre of a convex lens. The focal
A 2 B 3 length of the lens is 30 cm.
C 4 D 5

32 An object is placed at  18.0 cm from a convex


lens of focal length 20.0 cm.
What is the  characteristics of image ?
     Which of the following characteristics of the
A virtual, upright and magnified image is not correct when u is 20 cm ,30 cm , 
B real, inverted and magnified 40 cm and 70 cm from the lens?
C virtual ,upright and diminished u (cm) Characteristics of the 
D real, inverted and diminished                              image
A 20 Virtual and bigger
B 30 Virtual and bigger
33 Diagram shows ray diagram for a convex C 40 Real and smaller
lens. D 70 Real and smaller
38 Diagram shows the apparatus used to determine
the relationship between the object distance, u,
the image distance, v, and the focal length, f, of
a convex lens.

Which of the following is not true?

A The height of object is equal to the height of


the image
B The focal length of the lens is 80 cm
C The linear magnification is  1
D The image is a real image

The results are shown in graph below. . 

49
C 10 5
D 20 8

42 A lens has a focal length f.


What are the required conditions for the lens to
be used as a magnifying lens?
Type of lens Object distance
What is the value of p? 
A Concave Less than f
1 1 B Concave More than f
A f    B C Convex Less than f
2 f
C f     D    2f   D Convex More than f

39 The equation of a thin lens is given by the 43 Which of the following ray diagrams is correct 
1 1 1 for a magnifying glass?
equation : f  u  v

Based on the graph ,what is the focal length,f of


the lens?

A 0.02 cm B 0.04 cm
C 50.0 cm D 100.0 cm

40 A convex lens is used to produce a real,


magnified and inverted image. What is the
effect on the image produced when the upper
portion of the lens is covered by a coin ? 
    
     A The upright image is formed
B The diminished image is formed
C The size of the image is reduced 
D The  brightness of the image is reduced

41 Diagram shows an ant is seen through a 44 Which of the following pairs of lens are used to
magnifying glass. construct a telescope?

A A concave lens of focal length 3 cm and a


concave lens of focal length 120 cm
B A convex lens of focal length 3 cm and a
convex lens of focal length 120 cm
C A concave lens of focal length 100 cm and a
concave lens of focal length 120 cm
Which of the following combinations is true for
D A convex lens of focal length 100 cm and a
the magnifying lens ? 
convex lens of focal length 120 cm
Distance between ant Focal length
45 Two convex  lens lens W and lens X of focal
and magnifying glass of magnifying
lengths 40 cm and 5 cm respectively is used in
(cm) glass (cm)
a telescope.
A 5 10
Which of the following is true?
B 8 5

50
Objective Eyepiece Distance
lens between lens W
and X at normal
adjusment (cm)
A W X 35
B X W 35
C W X 45
D X W 45

46 A telescope has two convex lens of focal


length 5.0 cm and 50 c respectively.Which
of the following is true?

Linear Distance between lenses


magnification (cm)
A 250 45
B 250 55
C 10 45
D 10 55

47 The diagram shows the design of a simple


astronomical telescope.

49 Choose the most suitable combinations of the


lens that be used to construct a microscope.

Objective lens Eyepiece


A Concave lens of Concave lens of
Which characteristics of images formed by lens
focal length 3 cm focal length 120 cm
P and lens Q.
B Convex lens of Convex lens of
Charateristics of Charateristics of
focal length 3 cm focal length 120 cm
image P image Q
A virtual, upright virtual , upright
C Concave lens of Concave lens of
B real, inverted virtual , inverted focal length 3 cm focal length 5 cm
C virtual, inverted real, inverted
D real, upright real, inverted D Convex lens of Convex lens of
focal length 3 cm focal length 5 cm
48 Which arrangement of objective lens, Lo.
eyepiece, Le and position of image I, is correct
for a telescope at normal adjustment? 50 Which of the following is the correct positions
of the object and the final image of a compound
microscope at a normal adjusment?

51
Diagram 53
(a) State the phenomenon of light involved.
........................................................................
[ 1 mark ]
(b) The image of writing look bigger when
viewed through the water droplet.
(i) Why the image of the writing look bigger
..................................................................
..................................................................
[ 2 marks ]
(ii) State the two other characteristics of the
image of the writing.
..................................................................
[ 1 mark ]
51 What is the characteristics of image formed by
(c) What is the change size of the image of the
lens in a digital camera.
writing when the water droplet is replaced
with a transparent liquid droplet of a
A virtual, upright and magnified
greater optical density?
B real, inverted and magnified
..................................................................
C real, inverted and diminished
[ 1 mark ]
D virtual ,upright and diminished
(d) The water droplet is replaced with a lens
with focal length of 20 cm. The distance
between the writing and the centre of the
lens is 10 cm.It is observed that the image
of the writing is enlarged.
52 Diagram shows ray diagram of a camera.
(i) Name the type of the lens.
…………………………………….....
[ 1 mark ]
(ii) Calculate the distance of the image of the
writing from the optical centre of the
lens.

Based on diagram which of the following is


true?
A u<f B u=f [ 2 marks ]
C f < u < 2f D u > 2f (iii) In the space below sketch a ray
diagram to show  how the image is
53 Diagram 53 shows a water droplet on a piece formed.
of wax paper with writing on it.                                               

52
………………………………
[ 1 mark ]
(ii) Give one reason to your answer 
in (d) (i).
………………………………..................
[ 1 mark ]
(iii)What is the distance between lens X and
lens Y , when the telescope at a normal
adjusment.
..................................................................
[ 1 mark ]
[ 3 marks ] 55 Diagram 55 shows an object is placed in front
54 Diagram 54 shows an object, O with  height of a concave lens. F is the focal point of the
2 cm placed in front of a convex lens, X.The lens.
focal length of the convex lens X is 10 cm.

Diagram 54
Diagram 55
(a) In Diagram 54 ,complete the ray diagram to (a) What is meant by the focal point of a lens.
show how an image formed. ........................................................................
[ 3 marks ] ........................................................................
[ 1 mark ]
(b) State the characteristics of the image (b) In Diagram 55 ,complete the ray diagram to
formed. show how an image formed.
……………………………………................ [ 3 marks ]
[ 1 mark ] (c) When the object is placed at a distance 15
(c) When the object is placed at a distance 15 cm from the lens , a virtual image at a
cm from the lens X, calculate: distance 5 cm from the lens is formed.
Calculate the focal length of the lens.
(i) the image distance. 

[ 2 marks ] [ 2 marks ]
(ii) linear magnification. (d) What happen to the image when a screen is
placed behind the lens.
........................................................................
[ 2 marks ] [ 1 mark ]
(d) A leaf blown by a strong wind covered the 56 (a) Diagram 56 shows a magnifying glass used
upper portion ot the convex lens X.What by long - sighted person to read a book.
will happen to the size and brightness of the
image?
……………………………………………....
........................................................................
[ 2 marks ]
(e) The convex lens X and another convex lens
Y of focal length 50 cm are used to construct
a simple astronomical telescope.
(i) Which lens is suitable to be
objective lens and eyepiece?
Diagran 56
Objective lens:  (i) What type of lens is used for the
……………………………… magnifying glass.
.................................................................
Eyepiece:  ( 1 mark )
(ii) Give the reason for your answer in (a)(i).

53
................................................................. (i) Type of lenses:
( 1 mark ) ........................................................................
(iii) State the characteristics of the image Reason : 
formed. ………………………………........................
…………………………………….......... [2 marks] 
[ 1 mark ] (ii) Object distance, u0:
(b) In the space below sketch a ray ........................................................................
diagram to show  how the image is Reason : 
formed. ………………………………........................
                                 [2 marks] 
(ii) Distance, d:
........................................................................
Reason : 
………………………………........................
[2 marks] 
 (d)Based on the answer in (c), choose the most
suitable arrangement for a sinple compound
microscope.

…………………………………………....................
[1 mark] 
57 Diagram 57 shows a ray diagram of the
               convex lens with a focal length of 25 mm in a
[ 3 marks ] camera of smartphone in which the minimum
(c) The magnifying glass also known as a distance between the optical centre of the lens
simple microscope but a compound . and the sensor is equal to the focal length of the
compound microscope contains an objective lens.
lens and an eyepiece lens. The focal length
of the objective lens is fo and the focal length
of eyepiece lens is fe.
Table 56 shows three arrangement of a
compund microscope.

Type of lenses
Object Distance Diagram 57
distance, uo between (a) One of the characteristic of the image
Arran
from objective lens formed by the lens is inverted.
gement 
objective and eyepiece (i) State two others characteristics of the
lens lens, d image formed.
Objective lens is ........................................................................
convex lens ........................................................................
J uo<fo d < (fo+fe) [2 marks] 
and eyepiece is
concave lens (ii) The image that appears is vertical even
though the image is actually inverted.
Objective len is
State how the image was made
concave lens
K uo = fo d = (fo+fe) upright?
and eyepiece is
..................................................................
convex lens
[1 mark] 
Both are convex (b) (i) State the normal adjusment of the lens in
L lens for objective fo <uo<2fo d > (fo+fe) the camera.
lens and eyepiece ..................................................................
Table 56 [1 mark] 
Based on Table 56, state the suitable (ii) What is the minimum thickness of the
arrangement in order to build a simple smartphone.
compound microscope. Give one reason for the .................................................................
suitable arrangement.

54
(c) When h = 5 mm and u = 80 cm , calculate -Lenses use to produce the largest and
H. the brightest image
-The position of the lenses
-Handling the telescope
[ 10 marks ]
 [ 2 marks ] 59 Diagram 59.1 and Diagram  59.2 show the light 
58 Diagram 58.1 and Diagram 58.2 show the rays from a same person but in different object
parallel rays directed towards the convex distance , passing through the two identical
lenses J and K. Both the lenses produce real cameras with a lens.
images. F is the focal point for each lens.

Diagram 59.1
       
              Diagram 58.1

Diagram 59.2

(a) (i) What is meant by a lens? 


[1 mark]
Diagram 58.2
(ii) Using diagram 59.1 and 59.2 , compare
the object distance , the height of the
(a) (i) What is meant by focal length?
image and the focal length of the lens.
[1 mark] 
State the relationship between
(ii) Using Diagram 58.1 and Diagram 58.2,
- the object distance and the height of
compare the rays of light directed to the
the image
lens the thickness of the lenses and the
- the object distance and the linear
effects it has on the refracted rays to make a
magnification.
deduction regarding the relationship between
[5 marks]
the thickness of the lenses and their focal
(b) Diagram 59.3 shows a tool used to repair
length.
watches.
[5 marks] 
(b) Diagram 58.3 shows the ray diagram of
a compound microscope.

Diagram 59.3
Explain how the tool works.
      [4 marks]
(c) You are given the opportunity to choose two
lenses  to make a compound microscope
Explain your answer based on the following
aspects :
Diagram 58.3 - Charactristics of lenses used
Based on Diagram 58.3 - The arrangement of the lenses
(i) Explain the working principle of the  [10 marks]
microscope. 60 Diagram 60.1 shows the light from light bulb is
[4 marks]  directed to dark box (object) and a beaker filled
(ii) Suggest and explain how to modify the of water. The position of white screen is
microscope to be a telescope, based on adjusted until a sharp image formed on the
the following aspect: screen.

55
Diagram 60.2 shows the same procedure is (a) Stale one suitable inference.
repeated but with different distance between [1 mark] 
dark box and the beaker.   (b) State one suitable hypothesis.
[1 mark]
(c) With the use of apparatus such as a convex
lens, screen and other suitable apparatus,
describe one experiment to investigate the
hypothesis stated in (b).
In your description, state clearly the
Diagram 60.1 following:

  (i) The aim of the experiment.


(ii) The variables in the experiment.
(iii) The list of apparatus and materials.
  (iv) The arrangement of the apparatus.
(v) The procedure of the experiment which
Diagram 60.2 should include one method of
Based on the information and observation- controlling the manipulated variable
(a) Stale one suitable inference. and one method of measuring the
[1 mark]  responding variable.
  (b) State one suitable hypothesis.   (vi) The way to tabulate the data.
[1 mark]   (vii) The way to analyse the data.
(c) With the use of apparatus such as a convex [10 marks] 
lens, screen and other suitable apparatus,
describe one experiment to investigate the
hypothesis stated in (b).
In your description, state clearly the
following:
  (i) The aim of the experiment.
(ii) The variables in the experiment.
(iii) The list of apparatus and materials.
  (iv) The arrangement of the apparatus.
(v) The procedure of the experiment which
should include one method of
controlling the manipulated variable
and one method of measuring the
responding variable.
  (vi) The way to tabulate the data.
  (vii) The way to analyse the data.
[10 marks] 
LESSON 4 – FORMATION OF
61 Diagram 61 shows two convex lenses with 
its envelopes respectively.
IMAGE BY SPHERICAL
MIRROR
4.1.Introduction

A spherical mirror is an opaque material which


consists of smooth, shiny and curved surface.
For example , the front and back surfaces of steel
spoons can act as concave mirrors and convex
mirrors, respectively
Images formed by mirrors due to the phenomenon
of reflection of light.
Rays of light from objects at infinity ( distant 
objects) are parallel rays. 
There are two types of spherical mirror
Diagram 61 i.e.concave mirror and convex mirror..

Based on the information and observation-

56
Pole of mirror , P
4.2.The comparison of the field of view between
plane mirror , convex mirror and concave The centre point on  the spherical mirror
mirror.
Centre of curvature , C

The centre of sphere that produced the concave or


convex mirror

Radius of curvature, r

The distance between the pole, P and the 


centre of curvature, C.

Principal axis  ,PFC :

A line which passes 


through the centre of curvature ,C and the pole of a
curved mirror, P 

Focal point , F     

A point throughwhich all rays traveling parallel to


the principal axis converge to or appear to diverge
from after reflection by the mirror.

Focal length , f :

C: Centre of curvature The distance between the principal focus ,F and the


Angle of incidence,i = angle of reflection,r pole of the curved mirror, P.

Field of view convex mirror > plane mirror > Object distance , u
concave mirror.
Distance of the object from the pole of the mirror,P

4.3 Common terminology of reflection of light  Image distance , v


on a spherical mirror
Distance of the image  from the pole of the mirror,P
Concave mirror :
4.4 Relationship between f and r

r = 2f

4.5 Relationship curvature of mirror and f

Conve mirror : The more curved the mirror , the shorter its focal
length, f.

57
4.6 Characteristics of image formed by spherical
mirror by using ray diagrams.

The main ray of light in drawing a ray diagram


are:

Ray 1 :

The method to complete ray diagram for convex


mirror and concave mirror

1.Draw Ray I from the top of object.


2.Draw Ray 2 or Ray 3 from the top of the
object.
3.Extrapolate Ray1 and Ray2 or Ray 3
until intersect each other.
4.Draw image at the intersection point.
5.Draw arrow from object to the image

Example 1

Complete the following ray diagram and state the


characteristics of image formed
Ray 2 :

Solution

                               or

Ray 3:

Example 2

58
Complete the following ray diagram and state the (c) f < u < 2f  or f < u <  r ( Object O is between F
characteristics of image formed and C.

Solution

(d)u = 2f  or u = r ( Object ,O is at C)

or

Example 3

(a) Complete the following ray diagrams for


concave mirror (e) u > 2f  or u > r ( Object, O is beyond C )
(b) State the characteristics of image formed
(c)Complete the table below

(a) u < f  ( Object between F and P )

(f)  u = ∞ ( Object ,O very far from the lens)

(b) u = f ( Object, O is at F )

The distance object, u Characteristics of image

59
u < f (d) u = 2f  or u = r ( Object ,O is at C)
u    = f
f  < u <  2f
u  = 2f
u  > 2f
u=∞
Solution

(a) u < f  ( Object between F and P )

Characteristics of image  : 
..................................................................................
..................................................................................
(e) u > 2f  or u > r ( Object, O is beyond C )

Characteristics of image  : 
..................................................................................
..................................................................................

(b) u = f ( Object, O is at F )

Characteristics of image  : 
..................................................................................
..................................................................................
F  u = ∞ ( Object ,O very far from the lens)

Characteristics of image  : 
..................................................................................
..................................................................................

(c) f < u < 2f  or f < u <  r ( Object O is between F


and C. Characteristics of image  : 
..................................................................................
..................................................................................

The distance object, u Characteristics of image


u < f
u    = f
f  < u <  2f
u  = 2f
u  > 2f
u=∞
Characteristics of image  : 
..................................................................................
..................................................................................

60
Example 4

(a) Complete the following ray diagrams for


convex mirror Solution
(b) State the characteristics of image formed
(c)Complete the table below (a) u < f  ( Object between F and P )
  
(a) u < f  ( Object between F and P )

Characteristics of image  : 
..................................................................................
...................................................................................
(b) f < u < 2f  or f < u <  r ( Object O is between F
and C) (b) f < u < 2f  or f < u <  r ( Object O is between F
and C)

Characteristics of image  : 
The object distance, u Characteristics of image ..................................................................................
u < f ................................................................................
f  <u <  2f
The object distance, u Characteristics of image
u < f
f  <u <  2f

61
4.7 Uses of the concave mirror and convex
mirror Example 6

4.7.1Concave mirror for shaving mirror ,a Complete ray diagram for the concave parabolic
make-up mirror and for dentist mirror mirror in a torchlight as shown below and state the
Function To magnify image of an characteristics of the final image.
object
Structure Used a concave mirror
with a large radius of
curvature. 
Normal adjusment u<f
Ray diagram ?
Characteris tics of final ?
image
Example 5

Complete ray diagram for the dentist mirror


as shown below and state the characteristics
of the final image.

Solution

Solution

4.7.3 Convex mirror at sharp corner of a


road or hung in a supermarkets.

Function To produced upright and


diminished image and
widen the field of view
Structure Convex mirror with a
4.7.2Reflector of in torch light and in car’s large diameter.
headlamp ( concave parabolic mirror) Normal adjusment The mirror is placed at
high place and the
object is located in field
Function To produce parallel ray of view of the mirror
of light ( at infinity) Ray diagram ?
Structure Concave parabolic Characteris tics of ?
mirror is used.  final image
Normal adjusment u =f
Ray diagram ?
Characteris tics of ?
final image

62
Example 7

(a) Complete ray diagram for a convex mirror Solution


in Diagram (a) and state the characteristics
of the final image (a)
(b) Complete ray diagram in Diagram (b) to
show the field view of a convex mirror.

(b)
Diagram (a)

Diagram (b)

63
TUTORIAL 4
1 The image produced by a miror is caused by 
the

A total internal reflection of light


B diffraction of light
C refraction of light A 1 B 2
D reflection of light C 3 D 4

2 A mirror with reflecting surface curved 6 Which diagram shows the correct reflection of
outwards is called light from a convex mirror?
[F is the focal point ; C is centre of
A Curved mirror curvature]
B Parabolic mirror
C Concave mirror
D Convex mirror

3 The centre of sphere of which the reflecting


surface of a spherical mirror is a part is called

A Pole of mirror
B Focal point of mirror
C Optical centre of mirror
D Centre of curvature of mirror

4 Diagram shows the parallel rays of light are


reflected by a concave mirror.

7 Which ray diagram for a concave mirror is


drawn correctly?
[C = centre of curvature of mirror, F = focal
point of mirror]

Based on diagram what is the radius of


curvature of the concave mirror.

A 2L B L
L L
C D
2 4

5 The diagram shows four light rays A, B, 


C and D from an object, O and is reflected by a
concave mirror  to form an image I. Which of
the light ray is not correct? 

64
A parallel
B diverged
C converged
D converged then diverged

10 Diagram shows an object placed on the centre


of curvature, C of a concave mirror

Which of the following image is formed?

11 Diagram shows  a candle placed in front of a 


8 Diagram shows an object placed in front of a
concave mirror of focal length, f.
concave mirror. The distance of the object is
less than the focal length,f, of the mirror.

What are the characteristics of the image


formed? 
The image formed is
A Real, inverted, magnified
B Real, inverted, diminished
A real, upright and magnified
C Virtual, upright, magnified
B real, inverted and  diminished
D Virtual, inverted, diminished
C real, inverted and magnified
D virtual, upright and diminished
9 Diagram shows an object placed on the focal
point , F of a concave mirror
12 An object is placed at a distance 18 cm from a
concave mirror of  focal length 10 cm. What is
the characteristics of the image formed? 

A real , upright  and diminished


B real , inverted  and magnified
C virtual , inverted and magnified  
D virtual , upright  and diminished

13 A  concave mirror has a focal length 15 cm. The


The reflected rays are image formed by the mirror is virtual, upright
and magnified. What is the object distance?

65
A 8 cm B 15 cm
C 20 cm D 30 cm

14 An object of height 6 cm is placed at a


distance 30 cm in front of a concave mirror of
focal length 15 cm.
What is the height of the image formed?

A 3 cm B 6 cm
B 12 cm D 24 cm

15 A parallel rays of light from distant object is


reflected 20 cm in front of a concave mirror.
When a candle is placed at a distance 45 cm
from the concave mirror , what are the
characteristics of the image of candle formed

A magnified , virtual and inverted 


B magnified , real, and upright 
C diminished ,virtual and upright
D diminished, real and inverted  19 Which of the following is the right reason for
replacing a plane mirror are used as rear- view
16 The image formed by a convex mirror always mirrors in motor vehicles  with a convex
mirror ? 
A magnified , virtual and inverted 
B magnified , real, and upright  A To shine the object
C diminished ,virtual and upright B To widen the field of view
D diminished, real and inverted  C To produce a brighter image
D To produce a sharper  image
17 Diagram shows an object is placed in front of
a convex mirror. 20 Diagram shows of a parabolic mirror in a torch
[ C is the centre of curvature ]. .light

The light bulb in the torch light is positioned at


the focal point of the mirror so that

A light is reflected to focal point


B light is reflected to all directions
Image of the object formed at C light is reflected as parallel beam
D light bulb is reflected and spread out
A I B II
C III D IV 21 Diagram 21 shows image of a woman in a
spoon when she holds the spoon.
18 Which of the following is the application of a
convex mirror except

66
Diagram 22

(a) Based on Diagram 22

Diagram 21 (i) state the type of mirror is used.


…………………………........................
(a) Tick (√) the correct answer in the box [1 mark]
provided (ii) state one advantage the mirror is used
compare to the plane mirror.
The spoon acts as a ………………………………………......
[1 mark]
concave mirror (b) On the space below, draw a ray diagram
to show how the image formed by the
convex mirror makeup mirror.

[ 1 mark ]
(b) State the characteristics of the image
formed
..................................................................
..................................................................
[ 1 mark ]
(c) On the space below, draw a ray diagram
to show how the image of the woman
formed by the spoon.

[3 marks]
(ii) State the characteristics of the image
formed.
……………………………………..........
[1 mark]
(c) (i) What happen to the size of the image
when the curvature of the is increased.
…………………………………..............
[1 mark]
[ 3 marks ] (ii) Give the reason for your answer in
22 Diagram 22 shows a curved mirror used as a (c)(i).
makeup mirror. …………………………………..............
[1 mark]

23 Diagram 23.1 shows two cars P and Q, moving


in opposite directions, through a sharp bend. A
mirror is placed on the X.

67
Diagram 23.1

(a) Diagram 23.2 shows an incomplete ray Diagram 23.3


diagram when a plane mirror is placed in the
X position. (i) Name the curved mirror.
…..…………………………………........
[1 mark]
(ii) Complete the ray diagram in Diagram
23.3
[2 marks]

(iii) Based on your answer in b (ii), how can


the curved mirror solve the problem in
(a) (iii)?
…. …………………………………........
……………………………………..........
[1 mark]
(c) The image characteristics produced by the
Diagram 23..2 curved mirror in Figure 23.3 are diminished,
virtual and upright
(i) Complete the ray diagram in Diagram
23.2 (i) What happens to the characteristic of the
[2 marks] image as the focal length of the curved
mirror increases?
(ii) State the light phenomenon involved in ………………………………………......
(a) (i). [1 mark]
………………………………………...... (ii) Give the reason for your answer in
[1 mark] (c)(i).
………………………………………......
(iii) Based on your answer in (a) (i), state the [1 mark]
problem faced by driver P.
………………………………………...... 24  An object is placed at a distance ,u from a 
………………………………………...... concave mirror, M. Table 24 shows the
[1 mark] characteristics of image , when the distance  u is
(b) Diagram 23.3 shows an incomplete ray varied.
diagram when a curved mirror is positioned
to replace the plane mirror in Diagram 23.2. Object Characteristics of image
The curve miior is used to solve the distance,
problem in (a) (iii). u ( cm)
10
20
30
40
Infinity Diminished ,inverted,real
,formed at 20 cm in front of the
mirror
Table 24

68
25 Diagram 25.1 and Diagram 25.2 show the ray
(a) Name the light phenomenon that is diagrams of a light bulb is placed at two
involved  to form the image by the different distances in front of a concave mirror.
mirror. 
………………………………………......
[1 mark]
(b) Using the information in the table,
determine the focal length of the mirror
M. 
…………………………………………..
[1 mark]
(c) Give the reason for your answer in (b).
………………………………………......
[1 mark]
(d) Complete Table 24
[2 marks]

(e) An object is placed at a distance 35 cm


from the concave mirror M.
(i) Draw a ray diagram in the space (a) What is the characteristic of the image
below to show the formation of the formed in Diagram 25.1 ?
image. Tick (√) the correct answer in the box
provided.

real 

virtual 
[ 1 mark ]
(b) Observed Diagram 25.1 and Diagram 
25.2.
  (i) Compare the object distance.
..................................................................
[1 mark] 
(ii) Compare the image distance.
..................................................................
[1 mark] 
(iii) Compare the size of the image. 
..................................................................
[1 mark] 
[3 marks] (c) Based on the answer in (b)
(i) state the relationship between object
(ii) What is the characteristics of the distance and the image distance.
image? .................................................................
…………………………………….... [1 mark] 
[1 mark] (ii) state the relationship between object
(iii)What is the characteristics of the distance and the size of the image. 
image when the concave mirror M is ..................................................................
replaced by a convex mirror of focal [1 mark] 
length same as the mirror M. (d) Diagram 25.3 shows a parabolic dish that is
…………………………………….... used to receive signals from a satellite
[1 mark] television station.

Diagram 25.3

69
(i) Where should the receiver be placed to
obtain the strongest signal?
..................................................................
[1 mark] 
(ii) Give one reason for your answer in
(d)(i).
..................................................................
[1 mark] 
26 Diagram 26.1 and Diagram 26.2  show the
rays of light directed toward the curved
mirrors, J and K. CP is the radius of curvature,
and F is the focal point of the mirrors.

          Diagram 26.1                  Diagram 26.2

(a) (i) What is meant by focal length of a


curved mirror?
[ 1 mark ]
(ii) Using Diagram 26.1 and Diagram 26.2 ,
compare the type of mirror , the rays of
light directed to the mirror, the curvature
of the mirrors and their focal lengths.
Hence relate the curvature of the mirrors
and the focal lengths.
[ 5 marks ]
(b) Diagram 26.3 shows  the image of a
patient’s teeth seen in  a mirror used by a
dentist. 

Diagram 26.3

Explain how the mirror produced image of


the teeth?
[ 4 marks ]
(c) You are required to design a make-up
mirror. The make-up mirror that is included
with other items in a handbag The  make-up
mirror can be used during the day or night.
Explain your answer based on the following
aspects :
-  Charactristics of  mirror used
- The casing of the make-up mirror
-  Extra feature of the make-up mirror
 [10 marks]
 

70

You might also like